Respiratory NCLEX questions

¡Supera tus tareas y exámenes ahora con Quizwiz!

What is the most likely cause of a tension pneumothorax? Answers: A. Chest injury B. Untreated pneumonia C. Sarcoidosis D. Overdose of asthma medication

A A tension pneumothorax is usually caused by blunt trauma.

A male client with Guillain-Barré syndrome develops respiratory acidosis as a result of reduced alveolar ventilation. Which combination of arterial blood gas (ABG) values confirms respiratory acidosis? A. pH, 5.0; PaCO2 30 mm Hg B. pH, 7.40; PaCO2 35 mm Hg C. pH, 7.35; PaCO2 40 mm Hg D. pH, 7.25; PaCO2 50 mm Hg

D

Before weaning a male client from a ventilator, which assessment parameter is most important for the nurse to review? A. Fluid intake for the last 24 hours B. Baseline arterial blood gas (ABG) levels C. Prior outcomes of weaning D. Electrocardiogram (ECG) results

D

Why would a Venturi mask be used? Choose the answer that is most correct Answers: A. In treating influenza B. To lower risk of infection when treating certain clients C. To provide reactive support D. When high flow oxygen is needed

D A Venturi mask is used to provide high flow oxygen support.

A patient is being discharged from the emergency department after being treated for epistaxis. In teaching the family first aid measures in the event the epistaxis would recur, what measures should the nurse suggest (select all that apply)? Tilt patient's head backwards. Apply ice compresses to the nose. Tilt head forward while lying down. Pinch the entire soft lower portion of the nose. Partially insert a small gauze pad into the bleeding nostril.

First aid measures to control epistaxis include placing the patient in a sitting position, leaning forward. Pinching the soft lower portion of the nose or inserting a small gauze pad into the bleeding nostril should stop the bleeding within 15 minutes. Tilting the head back or forward does not stop the bleeding, but rather allows the blood to enter the nasopharynx, which could result in aspiration or nausea/vomiting from swallowing blood. Lying down also will not decrease the bleeding.

The nurse evaluates that a patient is experiencing the expected beneficial effects of ipratropium (Atrovent) after noting which assessment finding? Decreased respiratory rate Increased respiratory rate Increased peak flow readings Decreased sputum production

Ipratropium is a bronchodilator that should result in increased peak expiratory flow rates (PEFRs).

The patient had abdominal surgery yesterday. Today the lung sounds in the lower lobes have decreased. The nurse knows this could be due to what occurring? Pain Atelectasis Pneumonia Pleural effusion

Postoperatively there is an increased risk for atelectasis from anesthesia as well as restricted breathing from pain. Without deep breathing to stretch the alveoli, surfactant secretion to hold the alveoli open is not promoted. Pneumonia will occur later after surgery. Pleural effusion occurs because of blockage of lymphatic drainage or an imbalance between intravascular and oncotic fluid pressures, which is not expected in this case.

A patient with recurrent shortness of breath has just had a bronchoscopy. What is a priority nursing action immediately following the procedure? Monitor the patient for laryngeal edema. Assess the patient's level of consciousness. Monitor and manage the patient's level of pain. Assess the patient's heart rate and blood pressure.

Priorities for assessment are the patient's airway and breathing, both of which may be compromised after bronchoscopy by laryngeal edema. These assessment parameters supersede the importance of loss of consciousness (LOC), pain, heart rate, and blood pressure, although the nurse should also be assessing these.

Which task can the registered nurse (RN) delegate to unlicensed assistive personnel (UAP) in the care of a stable patient who has a tracheostomy? Assessing the need for suctioning Suctioning the patient's oropharynx Assessing the patient's swallowing ability Maintaining appropriate cuff inflation pressure

Providing the individual has been trained in correct technique, UAP may suction the patient's oropharynx. Assessing the need for suctioning should be performed by an RN or licensed practical nurse, whereas swallowing assessment and the maintenance of cuff inflation pressure should be performed solely by the RN.

When caring for a patient who is 3 hours postoperative laryngectomy, what is the nurse's highest priority assessment? Patient comfort Airway patency Incisional drainage Blood pressure and heart rate

Remember the ABCs with prioritization. Airway patency is always the highest priority and is essential for a patient undergoing surgery surrounding the upper respiratory system. Comfort, drainage, and vital signs follow the ABCs in priority.

When planning appropriate nursing interventions for a patient with metastatic lung cancer and a 60-pack-per-year history of cigarette smoking, the nurse recognizes that the smoking has most likely decreased the patient's underlying respiratory defenses because of impairment of cough reflex. mucociliary clearance. reflex bronchoconstriction. ability to filter particles from the air.

Smoking decreases the ciliary action in the tracheobronchial tree, resulting in impaired clearance of respiratory secretions and particles, chronic cough, and frequent respiratory infections.

The nurse is assigned to care for a patient in the emergency department admitted with an exacerbation of asthma. The patient has received a β-adrenergic bronchodilator and supplemental oxygen. If the patient's condition does not improve, the nurse should anticipate what as the most likely next step in treatment? IV fluids Biofeedback therapy Systemic corticosteroids Pulmonary function testing

Systemic corticosteroids speed the resolution of asthma exacerbations and are indicated if the initial response to the β-adrenergic bronchodilator is insufficient. IV fluids may be used, but not to improve ventilation. Biofeedback therapy and pulmonary function testing may be used after recovery to assist the patient and monitor the asthma.

The nurse is caring for a male client who recently underwent a tracheostomy. The first priority when caring for a client with a tracheostomy is: A. helping him communicate B. keeping his airway patent C. encouraging him to perform activities of daily living D. preventing him from developing an infection

A

A client is on Advair (fluticasone and salmeterol) for asthma. What is an important piece of information they should know? Answers: A. That it can increase the risk of death B. That it contains a short acting beta agonist C. It can increase the risk of lactic acidosis D. It is best used only several times a week

A Advair is a combination of a long acting beta agonist and a steroid. It is effective at treating asthma and widely used, but some data indicates that it can increase the risk of death.

A pulse oximetry reading of a client is 88%. What action should the nurse typically perform? Answers: A. Notify physician B. Elevate head C. Assess for COPD or asthma D. Perform a patient history

A If the oximetry reading is 88%, the nurse should typically notify a physician. The details may depend on your facility.

A patient had an open reduction repair of a bilateral nasal fracture. The nurse plans to implement an intervention that focuses on both nursing and medical goals for this patient. Which intervention should the nurse implement? Apply an external splint to the nose. Insert plastic nasal implant surgically. Humidify the air for mouth breathing. Maintain surgical packing in the nose.

A goal that is common to nursing and medical management of a patient after rhinoplasty is to prevent the formation of a septal hematoma and potential infections resulting from a septal hematoma. Therefore the nurse helps to keep the nasal packing in the nose. The packing applies direct pressure to oozing blood vessels to stop postoperative bleeding. A medical goal includes realigning the fracture with an external or internal splint. The nurse helps maintain the airway by humidifying inspired air because the nose is unable to do so following surgery because it is swollen and packed with gauze.

The nurse determines that the patient understood medication instructions about the use of a spacer device when taking inhaled medications after hearing the patient state what as the primary benefit? "I will pay less for medication because it will last longer." "More of the medication will get down into my lungs to help my breathing." "Now I will not need to breathe in as deeply when taking the inhaler medications." "This device will make it so much easier and faster to take my inhaled medications."

A spacer assists more medication to reach the lungs, with less being deposited in the mouth and the back of the throat. It does not affect the cost or increase the speed of using the inhaler.

Which clinical manifestation should the nurse expect to find during assessment of a patient admitted with pneumonia? Hyperresonance on percussion Vesicular breath sounds in all lobes Increased vocal fremitus on palpation Fine crackles in all lobes on auscultation

A typical physical examination finding for a patient with pneumonia is increased vocal fremitus on palpation. Other signs of pulmonary consolidation include bronchial breath sounds, egophony, and crackles in the affected area. With pleural effusion, there may be dullness to percussion over the affected area.

A male client who weighs 175 lb (79.4 kg) is receiving aminophylline (Aminophyllin) (400 mg in 500 ml) at 50 ml/hour. The theophylline level is reported as 6 mcg/ml. The nurse calls the physician who instructs the nurse to change the dosage to 0.45 mg/kg/hour. The nurse should: A. Question the order because it's too low. B. Question the order because it's too high. C. Set the pump at 45 ml/hour. D. Stop the infusion and have the laboratory repeat the theophylline measurement.

A. A therapeutic theophylline level is 10 to 20 mcg/ml. The client is currently receiving 0.5 mg/kg/hour of aminophylline. Because the client's theophylline level is sub-therapeutic, reducing the dose (which is what the physician's order would do) would be inappropriate. Therefore, the nurse should question the order.

A female client with chronic obstructive pulmonary disease (COPD) takes anhydrous theophylline, 200 mg P.O. every 8 hours. During a routine clinic visit, the client asks the nurse how the drug works. What is the mechanism of action of anhydrous theophylline in treating a nonreversible obstructive airway disease such as COPD? A. It makes the central respiratory center more sensitive to carbon dioxide and stimulates the respiratory drive. B. It inhibits the enzyme phosphodiesterase, decreasing degradation of cyclic adenosine monophosphate, a bronchodilator. C. It stimulates adenosine receptors, causing bronchodilation. D. It alters diaphragm movement, increasing chest expansion and enhancing the lung's capacity for gas exchange.

A. Anhydrous theophylline and other methylxanthine agents make the central respiratory center more sensitive to CO2 and stimulate the respiratory drive. Inhibition of phosphodiesterase is the drug's mechanism of action in treating asthma and other reversible obstructive airway diseases — not COPD. Methylxanthine agents inhibit rather than stimulate adenosine receptors. Although these agents reduce diaphragmatic fatigue in clients with chronic bronchitis or emphysema, they don't alter diaphragm movement to increase chest expansion and enhance gas exchange.

After receiving an oral dose of codeine for an intractable cough, the male client asks the nurse, "How long will it take for this drug to work?" How should the nurse respond? A. In 30 minutes B. In 1 hour C. In 2.5 hours D. In 4 hours

A. Codeine's onset of action is 30 minutes. Its peak concentration occurs in about 1 hour; its half-life, in 2.5 hours; and its duration of action is 4 to 6 hours.

A female patient suffers adult respiratory distress syndrome as a consequence of shock. The patient's condition deteriorates rapidly, and endotracheal intubation and mechanical ventilation are initiated. When the high pressure alarm on the mechanical ventilator, alarm sounds, the nurse starts to check for the cause. Which condition triggers the high pressure alarm? A. Kinking of the ventilator tubing B. A disconnected ventilator tube C. An endotracheal cuff leak D. A change in the oxygen concentration without resetting the oxygen level alarm

A. Conditions that trigger the high pressure alarm include kinking of the ventilator tubing, bronchospasm or pulmonary embolus, mucus plugging, water in the tube, coughing or biting on endotracheal tube, and the patient's being out of breathing rhythm with the ventilator. A disconnected ventilator tube or an endotracheal cuff leak would trigger the low pressure alarm. Changing the oxygen concentration without resetting the oxygen level alarm would tigger the oxygen alarm.

A male client suffers adult respiratory distress syndrome as a consequence of shock. The client's condition deteriorates rapidly, and endotracheal (ET) intubation and mechanical ventilation are initiated. When the high-pressure alarm on the mechanical ventilator sounds, the nurse starts to check for the cause. Which condition triggers the high-pressure alarm? A. Kinking of the ventilator tubing B. A disconnected ventilator tube C. An ET cuff leak D. A change in the oxygen concentration without resetting the oxygen level alarm

A. Conditions that trigger the high-pressure alarm include kinking of the ventilator tubing, bronchospasm or pulmonary embolus, mucus plugging, water in the tube, coughing or biting on the ET tube, and the client's being out of breathing rhythm with the ventilator. A disconnected ventilator tube or an ET cuff leak would trigger the low-pressure alarm. Changing the oxygen concentration without resetting the oxygen level alarm would trigger the oxygen alarm.

. Nurse Lei caring for a client with a pneumothorax and who has had a chest tube inserted notes continues gentle bubbling in the suction control chamber. What action is appropriate? A. Do nothing, because this is an expected finding B. Immediately clamp the chest tube and notify the physician C. Check for an air leak because the bubbling should be intermittent D. Increase the suction pressure so that the bubbling becomes vigorous

A. Continuous gentle bubbling should be noted in the suction control chamber. Option b is incorrect. Chest tubes should only be clamped to check for an air leak or when changing drainage devices (according to agency policy). Option c is incorrect. Bubbling should be continuous and not intermittent. Option d is incorrect because bubbling should be gentle. Increasing the suction pressure only increases the rate of evaporation of water in the drainage system.

A male client comes to the emergency department complaining of sudden onset of diarrhea, anorexia, malaise, cough, headache, and recurrent chills. Based on the client's history and physical findings, the physician suspects legionnaires' disease. While awaiting diagnostic test results, the client is admitted to the facility and started on antibiotic therapy. What is the drug of choice for treating legionnaires' disease? A. Erythromycin (Erythrocin) B. Rifampin (Rifadin) C. Amantadine (Symmetrel) D. Amphotericin B (Fungizone)

A. Erythromycin is the drug of choice for treating legionnaires' disease. Rifampin may be added to the regimen if erythromycin alone is ineffective; however, it isn't administered first. Amantadine, an antiviral agent, and amphotericin B, an antifungal agent, are ineffective against legionnaires' disease, which is caused by bacterial infection.

For a male client with an endotracheal (ET) tube, which nursing action is most essential? A. Auscultating the lungs for bilateral breath sounds B. Turning the client from side to side every 2 hours C. Monitoring serial blood gas values every 4 hours D. Providing frequent oral hygiene

A. For a client with an ET tube, the most important nursing action is auscultating the lungs regularly for bilateral breath sounds to ensure proper tube placement and effective oxygen delivery. Although the other options are appropriate for this client, they're secondary to ensuring adequate oxygenation.

A male client admitted to an acute care facility with pneumonia is receiving supplemental oxygen, 2 L/minute via nasal cannula. The client's history includes chronic obstructive pulmonary disease (COPD) and coronary artery disease. Because of these history findings, the nurse closely monitors the oxygen flow and the client's respiratory status. Which complication may arise if the client receives a high oxygen concentration? A. Apnea B. Anginal pain C. Respiratory alkalosis D. Metabolic acidosis

A. Hypoxia is the main breathing stimulus for a client with COPD. Excessive oxygen administration may lead to apnea by removing that stimulus. Anginal pain results from a reduced myocardial oxygen supply. A client with COPD may have anginal pain from generalized vasoconstriction secondary to hypoxia; however, administering oxygen at any concentration dilates blood vessels, easing anginal pain. Respiratory alkalosis results from alveolar hyperventilation, not excessive oxygen administration. In a client with COPD, high oxygen concentrations decrease the ventilatory drive, leading to respiratory acidosis, not alkalosis. High oxygen concentrations don't cause metabolic acidosis.

The nurse in charge is teaching a client with emphysema how to perform pursed-lip breathing. The client asks the nurse to explain the purpose of this breathing technique. Which explanation should the nurse provide? A. It helps prevent early airway collapse. B. It increases inspiratory muscle strength. C. It decreases use of accessory breathing muscles. D. It prolongs the inspiratory phase of respiration.

A. Pursed-lip breathing helps prevent early airway collapse. Learning this technique helps the client control respiration during periods of excitement, anxiety, exercise, and respiratory distress. To increase inspiratory muscle strength and endurance, the client may need to learn inspiratory resistive breathing. To decrease accessory muscle use and thus reduce the work of breathing, the client may need to learn diaphragmatic (abdominal) breathing. In pursed-lip breathing, the client mimics a normal inspiratory-expiratory (I:E) ratio of 1:2. (A client with emphysema may have an I:E ratio as high as 1:4.)

A male client with pneumococcal pneumonia is admitted to an acute care facility. The client in the next room is being treated for mycoplasmal pneumonia. Despite the different causes of the various types of pneumonia, all of them share which feature? A. Inflamed lung tissue B. Sudden onset C. Responsiveness to penicillin. D. Elevated white blood cell (WBC) count

A. The common feature of all types of pneumonia is an inflammatory pulmonary response to the offending organism or agent. Although most types of pneumonia have a sudden onset, a few (such as anaerobic bacterial pneumonia and mycoplasmal pneumonia) have an insidious onset. Antibiotic therapy is the primary treatment for most types of pneumonia; however, the antibiotic must be specific for the causative agent, which may not be responsive to penicillin. A few types of pneumonia, such as viral pneumonia, aren't treated with antibiotics. Although pneumonia usually causes an elevated WBC count, some types, such as mycoplasmal pneumonia, don't.

Nurse Oliver is caring for a client immediately after removal of the endotracheal tube. The nurse reports which of the following signs immediately if experienced by the client? A. Stridor B. Occasional pink-tinged sputum C. A few basilar lung crackles on the right D. Respiratory rate 24 breaths/min

A. The nurse reports stridor to the physician immediately. This is a high-pitched, coarse sound that is heard with the stethoscope over the trachea. Stridor indicates airway edema and places the client at risk for airway obstruction. Options B, C, and D are not signs that require immediate notification of the physician.

A slightly obese female client with a history of allergy-induced asthma, hypertension, and mitral valve prolapse is admitted to an acute care facility for elective surgery. The nurse obtains a complete history and performs a thorough physical examination, paying special attention to the cardiovascular and respiratory systems. When percussing the client's chest wall, the nurse expects to elicit: A. Resonant sounds. B. Hyperresonant sounds. C. Dull sounds. D. Flat sounds.

A. When percussing the chest wall, the nurse expects to elicit resonant sounds — low-pitched, hollow sounds heard over normal lung tissue. Hyperresonant sounds indicate increased air in the lungs or pleural space; they're louder and lower pitched than resonant sounds. Although hyperresonant sounds occur in such disorders as emphysema and pneumothorax, they may be normal in children and very thin adults. Dull sounds, normally heard only over the liver and heart, may occur over dense lung tissue, such as from consolidation or a tumor. Dull sounds are thudlike and of medium pitch. Flat sounds, soft and high-pitched, are heard over airless tissue and can be replicated by percussing the thigh or a bony structure.

The nurse is caring for a patient with chronic obstructive pulmonary disorder (COPD) and pneumonia who has an order for arterial blood gases to be drawn. What is the minimum length of time the nurse should plan to hold pressure on the puncture site? 2 minutes 5 minutes 10 minutes 15 minutes

After obtaining blood for an arterial blood gas measurement, the nurse should hold pressure on the puncture site for 5 minutes by the clock to be sure that bleeding has stopped. An artery is an elastic vessel under much higher pressure than veins, and significant blood loss or hematoma formation could occur if the time is insufficient.

The nurse evaluates that nursing interventions to promote airway clearance in a patient admitted with COPD are successful based on which finding? Absence of dyspnea Improved mental status Effective and productive coughing PaO2 within normal range for the patient

Airway clearance is most directly evaluated as successful if the patient can engage in effective and productive coughing. Absence of dyspnea, improved mental status, and PaO2 within normal range for the patient show improved respiratory status but do not evaluate airway clearance.

The patient has an order for each of the following inhalers. Which one should the nurse offer to the patient at the onset of an asthma attack? Albuterol (Proventil) Salmeterol (Serevent) Beclomethasone (Qvar) Ipratropium bromide (Atrovent)

Albuterol is a short-acting bronchodilator that should be given initially when the patient experiences an asthma attack. Salmeterol (Serevent) is a long-acting β2-adrenergic agonist, which is not used for acute asthma attacks. Beclomethasone (Qvar) is a corticosteroid inhaler and not recommended for an acute asthma attack. Ipratropium bromide (Atrovent) is an anticholinergic agent that is less effective than β2-adrenergic agonists. It may be used in an emergency with a patient unable to tolerate short-acting β2-adrenergic agonists (SABAs).

The nurse determines that the patient is not experiencing adverse effects of albuterol (Proventil) after noting which patient vital sign? Pulse rate of 72/minute Temperature of 98.4° F Oxygen saturation 96% Respiratory rate of 18/minute

Albuterol is a β2-agonist that can sometimes cause adverse cardiovascular effects. These would include tachycardia and angina. A pulse rate of 72 indicates that the patient did not experience tachycardia as an adverse effect.

The nurse is scheduled to administer seasonal influenza vaccinations to the residents of a long-term care facility. What would be a contraindication to the administration of the vaccine to a resident? Hypersensitivity to eggs Age greater than 80 years History of upper respiratory infections Chronic obstructive pulmonary disease (COPD)

Although current vaccines are highly purified, and reactions are extremely uncommon, a hypersensitivity to eggs precludes vaccination because the vaccine is produced in eggs. Advanced age and a history of respiratory illness are not contraindications for influenza vaccination.

What is the priority nursing intervention in helping a patient expectorate thick lung secretions? Humidify the oxygen as able. Administer cough suppressant q4hr. Teach patient to splint the affected area. Increase fluid intake to 3 L/day if tolerated.

Although several interventions may help the patient expectorate mucus, the highest priority should be on increasing fluid intake, which will liquefy the secretions so that the patient can expectorate them more easily. Humidifying the oxygen is also helpful but is not the primary intervention. Teaching the patient to splint the affected area may also be helpful in decreasing discomfort but does not assist in expectoration of thick secretions.

When admitting a patient with a diagnosis of asthma exacerbation, the nurse will assess for what potential triggers (select all that apply)? Exercise Allergies Emotional stress Decreased humidity Upper respiratory infections

Although the exact mechanism of asthma is unknown, there are several triggers that may precipitate an attack. These include allergens, exercise, air pollutants, upper respiratory infections, drug and food additives, psychologic factors, and gastroesophageal reflux disease (GERD).

A 45-year-old man with asthma is brought to the emergency department by automobile. He is short of breath and appears frightened. During the initial nursing assessment, which clinical manifestation might be present as an early manifestation during an exacerbation of asthma? Anxiety Cyanosis Bradycardia Hypercapnia

An early manifestation during an asthma attack is anxiety because the patient is acutely aware of the inability to get sufficient air to breathe. He will be hypoxic early on with decreased PaCO2 and increased pH as he is hyperventilating. If cyanosis occurs, it is a later sign. The pulse and blood pressure will be increased.

When admitting a 45-year-old female with a diagnosis of pulmonary embolism, the nurse will assess the patient for which risk factors (select all that apply)? Obesity Pneumonia Malignancy Cigarette smoking Prolonged air travel

An increased risk of pulmonary embolism is associated with obesity, malignancy, heavy cigarette smoking, and prolonged air travel with reduced mobility. Other risk factors include deep vein thrombosis, immobilization, surgery within the last 3 months, oral contraceptives and hormone therapy, heart failure, pregnancy, and clotting disorders.

While ambulating a patient with metastatic lung cancer, the nurse observes a drop in oxygen saturation from 93% to 86%. Which nursing intervention is most appropriate based upon these findings? Continue with ambulation since this is a normal response to activity. Obtain a physician's order for arterial blood gas determinations to verify the oxygen saturation. Obtain a physician's order for supplemental oxygen to be used during ambulation and other activity. Move the oximetry probe from the finger to the earlobe for more accurate monitoring during activity.

An oxygen saturation level that drops below 90% with activity indicates that the patient is not tolerating the exercise and needs to use supplemental oxygen. The patient will need to rest to resaturate. ABGs or moving the probe will not be needed as the pulse oximeter was working at the beginning of the walk.

A male adult client is suspected of having a pulmonary embolus. A nurse assesses the client, knowing that which of the following is a common clinical manifestation of pulmonary embolism? A. Dyspnea B. Bradypnea C. Bradycardia D. Decreased respirations

Answer A. The common clinical manifestations of pulmonary embolism are tachypnea, tachycardia, dyspnea, and chest pain.

The patient is hospitalized with pneumonia. Which diagnostic test should be used to measure the efficiency of gas transfer in the lung and tissue oxygenation? Thoracentesis Bronchoscopy Arterial blood gases Pulmonary function tests

Arterial blood gases are used to assess the efficiency of gas transfer in the lung and tissue oxygenation as is pulse oximetry. Thoracentesis is used to obtain specimens for diagnostic evaluation, remove pleural fluid, or instill medication into the pleural space. Bronchoscopy is used for diagnostic purposes, to obtain biopsy specimens, and to assess changes resulting from treatment. Pulmonary function tests measure lung volumes and airflow to diagnose pulmonary disease, monitor disease progression, evaluate disability, and evaluate response to bronchodilators.

A patient whose tracheostomy was inserted 30 minutes ago is recovering in the postanesthesia recovery unit when he coughs and expels the tracheostomy tube. How should the nurse respond? Suction the tracheostomy opening. Maintain the airway with a sterile hemostat. Use an Ambu bag and mask to ventilate the patient. Insert the tracheostomy tube obturator into the stoma.

As long as the patient is not in acute respiratory distress after dislodging the tracheostomy tube, the nurse should use a sterile hemostat to maintain an open airway until a sterile tracheostomy tube can be reinserted into the tracheal opening. The tracheostomy is an open surgical wound that has not had time to mature into a stoma. If the patient is in respiratory distress, the nurse will use an Ambu bag and mask to ventilate the patient temporarily.

A female client with interstitial lung disease is prescribed prednisone (Deltasone) to control inflammation. During client teaching, the nurse stresses the importance of taking prednisone exactly as prescribed and cautions against discontinuing the drug abruptly. A client who discontinues prednisone abruptly may experience: A. hyperglycemia and glycosuria. B. acute adrenocortical insufficiency. C. GI bleeding. D. restlessness and seizures

B

A male client abruptly sits up in bed, reports having difficulty breathing and has an arterial oxygen saturation of 88%. Which mode of oxygen delivery would most likely reverse the manifestations? A. Simple mask B. Non-rebreather mask C. Face tent D. Nasal cannula

B

Nurse Mickey is administering a purified protein derivative (PPD) test to a homeless client. Which of the following statements concerning PPD testing is true? A. A positive reaction indicates that the client has active tuberculosis (TB). B. A positive reaction indicates that the client has been exposed to the disease. C. A negative reaction always excludes the diagnosis of TB D. The PPD can be read within 12 hours after the injection

B

The amount of air inspired and expired with each breath is called: A. tidal volume B. residual volume. C. vital capacity D. dead-space volume

B

Which of the following would be most appropriate for a male client with an arterial blood gas (ABG) of pH 7.5, PaCO2 26 mm Hg, O2 saturation 96%, HCO3 24 mEq/L, and PaO2 94 mm Hg? A. Administer a prescribed decongestant B. Instruct the client to breathe into a paper bag C. Offer the client fluids frequently D. Administer prescribed supplemental oxygen.

B

What is important to know about emphysema? Answers: A. It's a complication of asthma B. It's a form of COPD C. It's a respiratory condition caused by bacteria D. It can only be reversed within a year or two of onset

B Emphysema is a form of COPD. It is caused by a variety of issues, in particular smoking. It cannot be reversed.

What is a true statement about SARS? Answers: A. It is caused by a retrovirus B. Symptoms typically occur within a week of infection C.There were the last human cases in 2007 D. It is contagious, but not particularly fatal

B SARS is a contagious, coronavirus that appeared in 2003. It's important to know, but there have been no recent human cases. It is contagious and has high rate of complication. Symptoms usually appear quickly after infection.

Rhea, confused and short breath, is brought to the emergency department by a family member. The medical history reveals chronic bronchitis and hypertension. To learn more about the current respiratory problem, the doctor orders a chest x-ray and arterial blood gas (ABG) analysis. When reviewing the ABG report, the nurses sees many abbreviations. What does a lowercase "a" in ABG value present? A. Acid-base balance B. Arterial Blood C. Arterial oxygen saturation D. Alveoli

B. A lowercase "a" in an ABG value represents arterial blood. For instance, the abbreviation PaO2 refers to the partial pressure of oxygen in arterial blood. The pH value reflects the acid base balance in arterial blood. Sa02 indicates arterial oxygen saturation. An uppercase "A" represents alveolar conditions: for example, PA02 indicates the partial pressure of oxygen in the alveoli.

A male client abruptly sits up in bed, reports having difficulty breathing and has an arterial oxygen saturation of 88%. Which mode of oxygen delivery would most likely reverse the manifestations? A. Simple mask B. Non-rebreather mask C. Face tent D. Nasal cannula

B. A non-rebreather mask can deliver levels of the fraction of inspired oxygen (FIO2) as high as 100%. Other modes — simple mask, face tent, and nasal cannula — deliver lower levels of FIO2.

When caring for a male patient who has just had a total laryngectomy, the nurse should plan to: A. Encourage oral feeding as soon as possible B. Develop an alternative communication method C. Keep the tracheostomy cuff fully inflated D. Keep the patient flat in bed

B. A patient with a laryngectomy cannot speak, yet still needs to communicate. Therefore, the nurse should plan to develop an alternative communication method. After a laryngectomy, edema interferes with the ability to swallow and necessitates tube (enteral) feedings. To prevent injury to the tracheal mucosa, the nurse should deflate the tracheostomy cuff or use the minimal leak technique. To decrease edema, the nurse should place the patient in semi-fowler's position.

Dr. Jones prescribes albuterol sulfate (Proventil) for a patient with newly diagnose asthma. When teaching the patient about this drug, the nurse should explain that it may cause: A. Nasal congestion B. Nervousness C. Lethargy D. Hyperkalemia

B. Albuterol may cause nervousness. The inhaled form of the drug may cause dryness and irritation of the nose and throat, not nasal congestion; insomnia, not lethargy; and hypokalemia (with high doses), not hyperkalemia. Otther adverse effects of albuterol include tremor, dizziness, headache, tachycardia, palpitations, hypertension, heartburn, nausea, vomiting and muscle cramps.

Nurse Reese is caring for a client hospitalized with acute exacerbation of chronic obstructive pulmonary disease. Which of the following would the nurse expect to note on assessment of this client? A. Hypocapnia B. A hyperinflated chest noted on the chest x-ray C. Increased oxygen saturation with exercise D. A widened diaphragm noted on the chest x-ray

B. Clinical manifestations of chronic obstructive pulmonary disease (COPD) include hypoxemia, hypercapnia, dyspnea on exertion and at rest, oxygen desaturation with exercise, and the use of accessory muscles of respiration. Chest x-rays reveal a hyperinflated chest and a flattened diaphragm if the disease is advanced.

The nurse in charge formulates a nursing diagnosis of Activity intolerance related to inadequate oxygenation and dyspnea for a client with chronic bronchitis. To minimize this problem, the nurse instructs the client to avoid conditions that increase oxygen demands. Such conditions include: A. Drinking more than 1,500 ml of fluid daily. B. Being overweight. C. Eating a high-protein snack at bedtime. D. Eating more than three large meals a day.

B. Conditions that increase oxygen demands include obesity, smoking, exposure to temperature extremes, and stress. A client with chronic bronchitis should drink at least 2,000 ml of fluid daily to thin mucus secretions; restricting fluid intake may be harmful. The nurse should encourage the client to eat a high-protein snack at bedtime because protein digestion produces an amino acid with sedating effects that may ease the insomnia associated with chronic bronchitis. Eating more than three large meals a day may cause fullness, making breathing uncomfortable and difficult; however, it doesn't increase oxygen demands. To help maintain adequate nutritional intake, the client with chronic bronchitis should eat small, frequent meals (up to six a day).

Nurse Reynolds caring for a client with a chest tube turns the client to the side, and the chest tube accidentally disconnects. The initial nursing action is to: A. Call the physician B. Place the tube in bottle of sterile water C. Immediately replace the chest tube system D. Place a sterile dressing over the disconnection site

B. If the chest drainage system is disconnected, the end of the tube is placed in a bottle of sterile water held below the level of the chest. The system is replaced if it breaks or cracks or if the collection chamber is full. Placing a sterile dressing over the disconnection site will not prevent complications resulting from the disconnection. The physician may need to be notified, but this is not the initial action.

While changing the tapes on a tracheostomy tube, the male client coughs and tube is dislodged. The initial nursing action is to: A. Call the physician to reinsert the tube B. Grasp the retention sutures to spread the opening C. Call the respiratory therapy department to reinsert the tracheotomy D. Cover the tracheostomy site with a sterile dressing to prevent infection

B. If the tube is dislodged accidentally, the initial nursing action is to grasp the retention sutures and spread the opening. If agency policy permits, the nurse then attempts immediately to replace the tube. Covering the tracheostomy site will block the airway. Options A and C will delay treatment in this emergency situation.

On auscultation, which finding suggests a right pneumothorax? A. Bilateral inspiratory and expiratory crackles B. Absence of breaths sound in the right thorax C. Inspiratory wheezes in the right thorax D. Bilateral pleural friction rub.

B. In pneumothorax, the alveoli are deflated and no air exchange occurs in the lungs. Therefore, breath sounds in the affected lung field are absent. None of the other options are associated with pneumothorax. Bilateral crackles may result from pulmonary congestion, inspiratory wheezes may signal asthma, and a pleural friction rub may indicate pleural inflammation.

A female adult client has a tracheostomy but doesn't require continuous mechanical ventilation. When weaning the client from the tracheostomy tube, the nurse initially should plug the opening in the tube for: A. 15 to 60 seconds. B. 5 to 20 minutes. C. 30 to 40 minutes. D. 45 to 60 minutes.

B. Initially, the nurse should plug the opening in the tracheostomy tube for 5 to 20 minutes, then gradually lengthen this interval according to the client's respiratory status. A client who doesn't require continuous mechanical ventilation already is breathing without assistance, at least for short periods; therefore, plugging the opening of the tube for only 15 to 60 seconds wouldn't be long enough to reveal the client's true tolerance to the procedure. Plugging the opening for more than 20 minutes would increase the risk of acute respiratory distress because the client requires an adjustment period to start breathing normally.

A nurse is caring for a male client with emphysema who is receiving oxygen. The nurse assesses the oxygen flow rate to ensure that it does not exceed: A. 1 L/min B. 2 L/min C. 6 L/min D. 10 L/min

B. Oxygen is used cautiously and should not exceed 2 L/min. Because of the long-standing hypercapnia that occurs in emphysema, the respiratory drive is triggered by low oxygen levels rather than increased carbon dioxide levels, as is the case in a normal respiratory system.

A black client with asthma seeks emergency care for acute respiratory distress. Because of this client's dark skin, the nurse should assess for cyanosis by inspecting the: A. Lips. B. Mucous membranes. C. Nail beds. D. Earlobes.

B. Skin color doesn't affect the mucous membranes. The lips, nail beds, and earlobes are less reliable indicators of cyanosis because they're affected by skin color.

A male client is asking the nurse a question regarding the Mantoux test for tuberculosis. The nurse should base her response on the fact that the: A. Area of redness is measured in 3 days and determines whether tuberculosis is present. B. Skin test doesn't differentiate between active and dormant tuberculosis infection. C. Presence of a wheal at the injection site in 2 days indicates active tuberculosis. D. Test stimulates a reddened response in some clients and requires a second test in 3 months.

B. The Mantoux test doesn't differentiate between active and dormant infections. If a positive reaction occurs, a sputum smear and culture as well as a chest X-ray are necessary to provide more information. Although the area of redness is measured in 3 days, a second test may be needed; neither test indicates that tuberculosis is active. In the Mantoux test, an induration 5 to 9 mm in diameter indicates a borderline reaction; a larger induration indicates a positive reaction. The presence of a wheal within 2 days doesn't indicate active tuberculosis.

An oxygen delivery system is prescribed for a male client with chronic obstructive pulmonary disease to deliver a precise oxygen concentration. Which of the following types of oxygen delivery systems would the nurse anticipate to be prescribed? A. Face tent B. Venturi mask C. Aerosol mask D. Tracheostomy collar

B. The Venturi mask delivers the most accurate oxygen concentration. It is the best oxygen delivery system for the client with chronic airflow limitation because it delivers a precise oxygen concentration. The face tent, aerosol mask, and tracheostomy collar are also high-flow oxygen delivery systems but most often are used to administer high humidity.

A nurse is caring for a male client with acute respiratory distress syndrome. Which of the following would the nurse expect to note in the client? A. Pallor B. Low arterial PaO2 C. Elevated arterial PaO2 D. Decreased respiratory rate

B. The earliest clinical sign of acute respiratory distress syndrome is an increased respiratory rate. Breathing becomes labored, and the client may exhibit air hunger, retractions, and cyanosis. Arterial blood gas analysis reveals increasing hypoxemia, with a PaO2 lower than 60 mm Hg.

A male patient has a sucking stab wound to the chest. Which action should the nurse take first? A. Drawing blood for a hematocrit and hemoglobin level B. Applying a dressing over the wound and taping it on three sides C. Preparing a chest tube insertion tray D. Preparing to start an I.V. line

B. The nurse immediately should apply a dressing over the stab wound and tape it on three sides to allow air to escape and to prevent tension pneumothorax (which is more life-threatening than an open chest wound). Only after covering and taping the wound should the nurse draw blood for laboratory tests, assist with chest tube insertion, and start an I.V. line.

Nurse Maureen has assisted a physician with the insertion of a chest tube. The nurse monitors the client and notes fluctuation of the fluid level in the water seal chamber after the tube is inserted. Based on this assessment, which action would be appropriate? A. Inform the physician B. Continue to monitor the client C. Reinforce the occlusive dressing D. Encourage the client to deep-breathe

B. The presence of fluctuation of the fluid level in the water seal chamber indicates a patent drainage system. With normal breathing, the water level rises with inspiration and falls with expiration. Fluctuation stops if the tube is obstructed, if a dependent loop exists, if the suction is not working properly, or if the lung has reexpanded. Options A, C, and D are incorrect.

The nurse assesses a male client's respiratory status. Which observation indicates that the client is experiencing difficulty breathing? A. Diaphragmatic breathing B. Use of accessory muscles C. Pursed-lip breathing D. Controlled breathing

B. The use of accessory muscles for respiration indicates the client is having difficulty breathing. Diaphragmatic and pursed-lip breathing are two controlled breathing techniques that help the client conserve energy.

An emergency room nurse is assessing a male client who has sustained a blunt injury to the chest wall. Which of these signs would indicate the presence of a pneumothorax in this client? A. A low respiratory rate B. Diminished breath sounds C. The presence of a barrel chest D. A sucking sound at the site of injury

B. This client has sustained a blunt or a closed chest injury. Basic symptoms of a closed pneumothorax are shortness of breath and chest pain. A larger pneumothorax may cause tachypnea, cyanosis, diminished breath sounds, and subcutaneous emphysema. Hyperresonance also may occur on the affected side. A sucking sound at the site of injury would be noted with an open chest injury.

A nurse is preparing to obtain a sputum specimen from a male client. Which of the following nursing actions will facilitate obtaining the specimen? A. Limiting fluid B. Having the client take deep breaths C. Asking the client to spit into the collection container D. Asking the client to obtain the specimen after eating

B. To obtain a sputum specimen, the client should rinse the mouth to reduce contamination, breathe deeply, and then cough into a sputum specimen container. The client should be encouraged to cough and not spit so as to obtain sputum. Sputum can be thinned by fluids or by a respiratory treatment such as inhalation of nebulized saline or water. The optimal time to obtain a specimen is on arising in the morning.

A nurse performs an admission assessment on a female client with a diagnosis of tuberculosis. The nurse reviews the result of which diagnosis test that will confirm this diagnosis? A. Bronchoscopy B. Sputum culture C. Chest x-ray D. Tuberculin skin test

B. Tuberculosis is definitively diagnosed through culture and isolation of Mycobacterium tuberculosis. A presumptive diagnosis is made based on a tuberculin skin test, a sputum smear that is positive for acid-fast bacteria, a chest x-ray, and histological evidence of granulomatous disease on biopsy.

The patient has been diagnosed with head and neck cancer. Along with the treatment for the cancer, what other treatment should the nurse expect? Nasal packing Epistaxis balloon Gastrostomy tube Peripheral skin care

Because 50% of patients with head and neck cancer are malnourished before treatment begins, many patients need enteral feeding via a gastrostomy tube because the effects of treatment make it difficult to take in enough nutrients orally, whether surgery, chemotherapy, or radiation is used. Nasal packing could be used with epistaxis or with nasal or sinus problems. Peripheral skin care would not be expected because it is not related to head and neck cancer.

The nurse is assisting a patient to learn self-administration of beclomethasone, two puffs inhaled every 6 hours. What should the nurse explain as the best way to prevent oral infection while taking this medication? Chew a hard candy before the first puff of medication. Rinse the mouth with water before each puff of medication. Ask for a breath mint following the second puff of medication. Rinse the mouth with water following the second puff of medication.

Because beclamethosone is a corticosteroid, the patient should rinse the mouth with water following the second puff of medication to reduce the risk of fungal overgrowth and oral infection.

The nurse is caring for a patient admitted to the hospital with pneumonia. Upon assessment, the nurse notes a temperature of 101.4° F, a productive cough with yellow sputum, and a respiratory rate of 20. Which nursing diagnosis is most appropriate based upon this assessment? Hyperthermia related to infectious illness Ineffective thermoregulation related to chilling Ineffective breathing pattern related to pneumonia Ineffective airway clearance related to thick secretions

Because the patient has spiked a temperature and has a diagnosis of pneumonia, the logical nursing diagnosis is hyperthermia related to infectious illness. There is no evidence of a chill, and her breathing pattern is within normal limits at 20 breaths/minute. There is no evidence of ineffective airway clearance from the information given because the patient is expectorating sputum.

A 71-year-old patient is admitted with acute respiratory distress related to cor pulmonale. Which nursing intervention is most appropriate during admission of this patient? Perform a comprehensive health history with the patient to review prior respiratory problems. Complete a full physical examination to determine the effect of the respiratory distress on other body functions. Delay any physical assessment of the patient and review with the family the patient's history of respiratory problems. Perform a physical assessment of the respiratory system and ask specific questions related to this episode of respiratory distress.

Because the patient is having respiratory difficulty, the nurse should ask specific questions about this episode and perform a physical assessment of this system. Further history taking and physical examination of other body systems can proceed once the patient's acute respiratory distress is being managed.

When caring for a patient with chronic obstructive pulmonary disease (COPD), the nurse identifies a nursing diagnosis of imbalanced nutrition: less than body requirements after noting a weight loss of 30 lb. Which intervention should the nurse add to the plan of care for this patient? Order fruits and fruit juices to be offered between meals. Order a high-calorie, high-protein diet with six small meals a day. Teach the patient to use frozen meals at home that can be microwaved. Provide a high-calorie, high-carbohydrate, nonirritating, frequent feeding diet.

Because the patient with COPD needs to use greater energy to breathe, there is often decreased oral intake because of dyspnea. A full stomach also impairs the ability of the diaphragm to descend during inspiration, thus interfering with the work of breathing. For these reasons, the patient with COPD should eat six small meals per day taking in a high-calorie, high-protein diet, with non-protein calories divided evenly between fat and carbohydrate. The other interventions will not increase the patient's caloric intake.

After assisting at the bedside with a thoracentesis, the nurse should continue to assess the patient for signs and symptoms of what? Bronchospasm Pneumothorax Pulmonary edema Respiratory acidosis

Because thoracentesis involves the introduction of a catheter into the pleural space, there is a risk of pneumothorax. Thoracentesis does not carry a significant potential for causing bronchospasm, pulmonary edema, or respiratory acidosis.

The patient who had idiopathic pulmonary fibrosis had a bilateral lung transplantation. Now he is experiencing airflow obstruction that is progressing over time. It started with a gradual onset of exertional dyspnea, nonproductive cough, and wheezing. What are these manifestations signs of in the lung transplant patient? Pulmonary infarction Pulmonary hypertension Cytomegalovirus (CMV) Bronchiolitis obliterans (BOS)

Bronchiolitis obliterans (BOS) is a manifestation of chronic rejection and is characterized by airflow obstruction progressing over time with a gradual onset of exertional dyspnea, nonproductive cough, wheezing, and/or low-grade fever. Pulmonary infarction occurs with lack of blood flow to the bronchial tissue or preexisting lung disease. With pulmonary hypertension, the pulmonary pressures are elevated and can be idiopathic or secondarily due to parenchymal lung disease that causes anatomic or vascular changes leading to pulmonary hypertension. CMV pneumonia is the most common opportunistic infection 1 to 4 months after lung transplant.

A male client with pneumonia develops respiratory failure and has a partial pressure of arterial oxygen of 55 mm Hg. He's placed on mechanical ventilation with a fraction of inspired oxygen (FIO2) of 0.9. The nursing goal should be to reduce the FIO2 to no greater than: A. 0.21 B. 0.35 C. 0.5 D. 0.7

C

A client has been prescribed Montelukast (Singulair). What information provided by the nurse would NOT be correct? Answers: A. It is a leukotrine blocker B. An uncommon side effect is psychiatric issues C. If used, it should be taken at least 1 hour before exercise D. It can be used in a child as young as 2

C For exercise related issues, Singulair should be taken at least 2 hours before exercise. It is a leukotrine blocker for asthma that, recent data indicates, can have psychiatric issues.

What condition can the Haemophilus influenzae type B, or Hib, vaccine help prevent? Answers: A. Bronchitis B. SIDS C. Epiglottitis D. Pneumothorax

C The Hib vaccine is known to help prevent epiglottitis.

A client reports that he is exhausted during the day, wakes up during the night, and has headaches. What condition might the nurse suspect he has? Answers: A. Pneumonia B. COPD C. Sleep apnea D. Pleural effusion

C Waking up at night and exhaustion are signs of sleep apnea.

During an assessment of a 45-year-old patient with asthma, the nurse notes wheezing and dyspnea. The nurse interprets that these symptoms are related to what pathophysiologic change? a. Laryngospasm b. Pulmonary edema c. Narrowing of the airway d. Overdistention of the alveoli

C Narrowing of the airway by persistent but variable inflammation leads to reduced airflow, making it difficult for the patient to breathe and producing the characteristic wheezing. Laryngospasm, pulmonary edema, and overdistention of the alveoli do not produce wheezing.

A male patient's X-ray result reveals bilateral white-outs, indicating adult respiratory distress syndrome (ARDS). This syndrome results from: A. Cardiogenic pulmonary edema B. Respiratory alkalosis C. Increased pulmonary capillary permeability D. Renal failure

C. ARDS results from increased pulmonary capillary permeability, which leads to noncardiogenic pulmonary edema. In cardiogenic pulmonary edema, pulmonary congestion occurs secondary to heart failure. In the initial stage of ARDS, respiratory alkalosis may arise secondary to hyperventilation; however, it does not cause ARDS. Renal failure does not cause ARDS, either.

Nurse Oliver observes constant bubbling in the water-seal chamber of a closed chest drainage system. What should the nurse conclude? A. The system is functioning normally B. The client has a pneumothorax. C. The system has an air leak. D. The chest tube is obstructed.

C. Constant bubbling in the chamber indicates an air leak and requires immediate intervention. The client with a pneumothorax will have intermittent bubbling in the water-seal chamber. Clients without a pneumothorax should have no evidence of bubbling in the chamber. If the tube is obstructed, the nurse should notice that the fluid has stopped fluctuating in the water-seal chamber.

For a female patient with chronic obstructive pulmonary disease, which nursing intervention would help maintain a patent airway? A. Restricting fluid intake to 1,000 ml per day B. Enforcing absolute bed rest C. Teaching the patient how to perform controlled coughing D. Administering prescribe sedatives regularly and in large amounts

C. Controlled coughing helps maintain a patent airway by helping to mobilize and remove secretions. A moderate fluid intake (usually 2 L or more daily) and moderate activity help liquefy and mobilize secretions. Bed rest and sedatives may limit the patient's ability to maintain a patent airway, causing a high risk for infection from pooled secretions.

A female client must take streptomycin for tuberculosis. Before therapy begins, the nurse should instruct the client to notify the physician if which health concern occurs? A. Impaired color discrimination B. Increased urinary frequency C. Decreased hearing acuity D. Increased appetite

C. Decreased hearing acuity indicates ototoxicity, a serious adverse effect of streptomycin therapy. The client should notify the physician immediately if it occurs so that streptomycin can be discontinued and an alternative drug can be prescribed. The other options aren't associated with streptomycin. Impaired color discrimination indicates color blindness; increased urinary frequency and increased appetite accompany diabetes mellitus.

A nurse is suctioning fluids from a female client through an endotracheal tube. During the suctioning procedure, the nurse notes on the monitor that the heart rate is decreasing. Which if the following is the appropriate nursing intervention? A. Continue to suction B. Notify the physician immediately C. Stop the procedure and reoxygenate the client D. Ensure that the suction is limited to 15 seconds

C. During suctioning, the nurse should monitor the client closely for side effects, including hypoxemia, cardiac irregularities such as a decrease in heart rate resulting from vagal stimulation, mucosal trauma, hypotension, and paroxysmal coughing. If side effects develop, especially cardiac irregularities, the procedure is stopped and the client is reoxygenated.

A nurse is suctioning fluids from a male client via a tracheostomy tube. When suctioning, the nurse must limit the suctioning time to a maximum of: A. 1 minute B. 5 seconds C. 10 seconds D. 30 seconds

C. Hypoxemia can be caused by prolonged suctioning, which stimulates the pacemaker cells in the heart. A vasovagal response may occur, causing bradycardia. The nurse must preoxygenate the client before suctioning and limit the suctioning pass to 10 seconds.

Nurse Joy is caring for a client after a bronchoscopy and biopsy. Which of the following signs, if noticed in the client, should be reported immediately to the physician? A. Dry cough B. Hermaturia C. Bronchospasm D. Blood-streaked sputum

C. If a biopsy was performed during a bronchoscopy, blood-streaked sputum is expected for several hours. Frank blood indicates hemorrhage. A dry cough may be expected. The client should be assessed for signs of complications, which would include cyanosis, dyspnea, stridor, bronchospasm, hemoptysis, hypotension, tachycardia, and dysrhythmias. Hematuria is unrelated to this procedure.

A male client with chronic obstructive pulmonary disease (COPD) is recovering from a myocardial infarction. Because the client is extremely weak and can't produce an effective cough, the nurse should monitor closely for: A. Pleural effusion. B. Pulmonary edema. C. Atelectasis. D. Oxygen toxicity.

C. In a client with COPD, an ineffective cough impedes secretion removal. This, in turn, causes mucus plugging, which leads to localized airway obstruction — a known cause of atelectasis. An ineffective cough doesn't cause pleural effusion (fluid accumulation in the pleural space). Pulmonary edema usually results from left-sided heart failure, not an ineffective cough. Although many noncardiac conditions may cause pulmonary edema, an ineffective cough isn't one of them. Oxygen toxicity results from prolonged administration of high oxygen concentrations, not an ineffective cough.

The nurse is teaching a male client with chronic bronchitis about breathing exercises. Which of the following should the nurse include in the teaching? A. Make inhalation longer than exhalation. B. Exhale through an open mouth. C. Use diaphragmatic breathing. D. Use chest breathing.

C. In chronic bronchitis the diaphragm is flat and weak. Diaphragmatic breathing helps to strengthen the diaphragm and maximizes ventilation. Exhalation should be longer than inhalation to prevent collapse of the bronchioles. The client with chronic bronchitis should exhale through pursed lips to prolong exhalation, keep the bronchioles from collapsing, and prevent air trapping. Diaphragmatic breathing — not chest breathing — increases lung expansion.

Miriam, a college student with acute rhinitis sees the campus nurse because of excessive nasal drainage. The nurse asks the patient about the color of the drainage. In a acute rhinitis, nasal drainage normally is: A. Yellow B. Green C. Clear D. Gray

C. Normally, nasal drainage in acute rhinitis is clear. Yellow or green drainage indicates spread of the infection to the sinuses. Gray drainage may indicate a secondary infection.

For a patient with advance chronic obstructive pulmonary disease (COPD), which nursing action best promotes adequate gas exchange? A. Encouraging the patient to drink three glasses of fluid daily B. Keeping the patient in semi-fowler's position C. Using a high-flow venture mask to deliver oxygen as prescribe D. Administering a sedative, as prescribe

C. The patient with COPD retains carbon dioxide, which inhibits stimulation of breathing by the medullary center in the brain. As a result, low oxygen levels in the blood stimulate respiration, and administering unspecified, unmonitored amounts of oxygen may depress ventilation. To promote adequate gas exchange, the nurse should use a Venturi mask to deliver a specified, controlled amount of oxygen consistently and accurately. Drinking three glasses of fluid daily would not affect gas exchange or be sufficient to liquefy secretions, which are common in COPD. Patients with COPD and respiratory distress should be places in high-Fowler's position and should not receive sedatives or other drugs that may further depress the respiratory center.

Which phrase is used to describe the volume of air inspired and expired with a normal breath? A. Total lung capacity B. Forced vital capacity C. Tidal volume D. Residual volume

C. Tidal volume refers to the volume of air inspired and expired with a normal breath. Total lung capacity is the maximal amount of air the lungs and respiratory passages can hold after a forced inspiration. Forced vital capacity is the vital capacity performed with a maximally forced expiration. Residual volume is the maximal amount of air left in the lung after a maximal expiration.

After undergoing a left pneumonectomy, a female patient has a chest tube in place for drainage. When caring for this patient, the nurse must: A. Monitor fluctuations in the water-seal chamber B. Clamp the chest tube once every shift C. Encourage coughing and deep breathing D. Milk the chest tube every 2 hours

C. When caring for a patient who is recovering from a pneumonectomy, the nurse should encourage coughing and deep breathing to prevent pneumonia in the unaffected lung. Because the lung has been removed, the water-seal chamber should display no fluctuations. Reinflation is not the purpose of chest tube. Chest tube milking is controversial and should be done only to remove blood clots that obstruct the flow of drainage.

The nurse notices clear nasal drainage in a patient newly admitted with facial trauma, including a nasal fracture. What should the nurse do first? Test the drainage for the presence of glucose. Suction the nose to maintain airway clearance. Document the findings and continue monitoring. Apply a drip pad and reassure the patient this is normal.

Clear nasal drainage suggests leakage of cerebrospinal fluid (CSF). The drainage should be tested for the presence of glucose, which would indicate the presence of CSF. Suctioning should not be done. Documenting the findings and monitoring are important after notifying the health care provider. A drip pad may be applied, but the patient should not be reassured that this is normal.

What should the nurse inspect when assessing a patient with shortness of breath for evidence of long-standing hypoxemia? Chest excursion Spinal curvatures Respiratory pattern Fingernails and their base

Clubbing, a sign of long-standing hypoxemia, is evidenced by an increase in the angle between the base of the nail and the fingernail to 180 degrees or more, usually accompanied by an increase in the depth, bulk, and sponginess of the end of the finger.

T

Co

A patient with a history of tonsillitis complains of difficulty breathing. Which patient assessment data warrants emergency interventions by the nurse? Bilateral erythema of especially large tonsils Temperature 102.2° F, diaphoresis, and chills Contraction of neck muscles during inspiration β-hemolytic streptococcus in the throat culture

Contraction of neck muscles during inspiration indicates that the patient is using accessory muscles for breathing and is in serious respiratory distress. The reddened and enlarged tonsils indicate pharyngitis. The increased temperature, diaphoresis, and chills indicate an infection, which could be β-hemolytic streptococcus or fungal infection, but not an emergency situation for the patient.

Nursing assessment findings of jugular venous distention and pedal edema would be indicative of what complication of chronic obstructive pulmonary disease (COPD)? Acute respiratory failure Secondary respiratory infection Fluid volume excess resulting from cor pulmonale Pulmonary edema caused by left-sided heart failure

Cor pulmonale is a right-sided heart failure caused by resistance to right ventricular outflow resulting from lung disease. With failure of the right ventricle, the blood emptying into the right atrium and ventricle would be slowed, leading to jugular venous distention and pedal edema.

The nurse, who has administered a first dose of oral prednisone to a patient with asthma, writes on the care plan to begin monitoring for which patient parameters? Apical pulse Daily weight Bowel sounds Deep tendon reflexes

Corticosteroids such as prednisone can lead to weight gain. For this reason, it is important to monitor the patient's daily weight. The drug should not affect the apical pulse, bowel sounds, or deep tendon reflexes.

The school nurse is providing information to high school students about influenza prevention. What should the nurse emphasize in teaching to prevent the transmission of the virus (select all that apply)? Cover the nose when coughing. Obtain an influenza vaccination. Stay at home when symptomatic. Drink non-caffeinated fluids daily. Obtain antibiotic therapy promptly.

Covering the nose and mouth when coughing is an effective way to prevent the spread of the virus. Obtaining an influenza vaccination helps prevent the flu. Staying at home helps prevent direct exposure of others to the virus. Drinking fluids helps liquefy secretions but does not prevent influenza. Antibiotic therapy is not used unless the patient develops a secondary bacterial infection.

A client has a Mantoux test performed. A response of 5mm would indicate: A. The presence of respiratory allergies B. The early stages of COPD in an at risk patient C. A need for bronchodilator treatment in asthma D. Unlikely tuberculosis infection in a low-risk patient

D In a client with low-risk, a Mantoux test response of 5mm is not considered positive for tuberculosis. In someone who has HIV or recent tuberculosis contact, it is positive, however.

A male patient is admitted to the health care facility for treatment of chronic obstructive pulmonary disease. Which nursing diagnosis is most important for this patient? A. Activity intolerance related to fatigue B. Anxiety related to actual threat to health status C. Risk for infection related to retained secretions D. Impaired gas exchange related to airflow obstruction

D. A patient airway and an adequate breathing pattern are the top priority for any patient, making "impaired gas exchange related to airflow obstruction" the most important nursing diagnosis. The other options also may apply to this patient but less important.

Before administering ephedrine, Nurse Tony assesses the patient's history. Because of ephedrine's central nervous system (CNS) effects, it is not recommended for: A. Patients with an acute asthma attack B. Patients with narcolepsy C. Patients under age 6 D. Elderly patients

D. Ephedrine is not recommended for elderly patients, who are particularly susceptible to CNS reactions (such as confusion and anxiety) and to cardiovascular reactions (such as increased systolic blood pressure, coldness in the extremities, and anginal pain). Ephedrine is used for its bronchodilator effects with acute and chronic asthma and occasionally for its CNS stimulant actions for narcolepsy. It can be administered to children age 2 and older.

A client with Guillain-Barré syndrome develops respiratory acidosis as a result of reduced alveolar ventilation. Which combination of arterial blood gas (ABG) values confirms respiratory acidosis? A. pH, 5.0; PaCO2 30 mm Hg B. pH, 7.40; PaCO2 35 mm Hg C. pH, 7.35; PaCO2 40 mm Hg D. pH, 7.25; PaCO2 50 mm Hg

D. In respiratory acidosis, ABG analysis reveals an arterial pH below 7.35 and partial pressure of arterial carbon dioxide (PaCO2) above 45 mm Hg. Therefore, the combination of a pH value of 7.25 and a PaCO2 value of 50 mm Hg confirms respiratory acidosis. A pH value of 5.0 with a PaCO2 value of 30 mm Hg indicates respiratory alkalosis. Options B and C represent normal ABG values, reflecting normal gas exchange in the lungs.

Blessy, a community health nurse is conducting an educational session with community members regarding tuberculosis. The nurse tells the group that one of the first symptoms associated with tuberculosis is: A. Dyspnea B. Chest pain C. A bloody, productive cough D. A cough with the expectoration of mucoid sputum

D. One of the first pulmonary symptoms is a slight cough with the expectoration of mucoid sputum. Options A, B, and C are late symptoms and signify cavitation and extensive lung involvement.

A male adult patient on mechanical ventilation is receiving pancuronium bromide (Pavulon), 0.01 mg/kg I.V. as needed. Which assessment finding indicates that the patient needs another pancuronium dose? A. Leg movement B. Finger movement C. Lip movement D. Fighting the ventilator

D. Pancuronium, a nondepolarizing blocking agent, is used for muscle relaxation and paralysis. It assists mechanical ventilation by promoting encdotracheal intubation and paralyzing the patient so that the mechanical ventilator can do its work. Fighting the ventilator is a sign that the patient needs another pancuronium dose. The nurse should administer 0.01 to 0.02 mg/kg I.V. every 20 to 60 minutes. Movement of the legs, or lips has no effect on the ventilator and therefore is not used to determine the need for another dose.

A nurse instructs a female client to use the pursed-lip method of breathing and the client asks the nurse about the purpose of this type of breathing. The nurse responds, knowing that the primary purpose of pursed-lip breathing is to: A. Promote oxygen intake B. Strengthen the diaphragm C. Strengthen the intercostal muscles D. Promote carbon dioxide elimination

D. Pursed-lip breathing facilitates maximal expiration for clients with obstructive lung disease. This type of breathing allows better expiration by increasing airway pressure that keeps air passages open during exhalation. Options A, B, and C are not the purposes of this type of breathing.

After undergoing a thoracotomy, a male client is receiving epidural analgesia. Which assessment finding indicates that the client has developed the most serious complication of epidural analgesia? A. Heightened alertness B. Increased heart rate C. Numbness and tingling of the extremities D. Respiratory depression

D. Respiratory depression is the most serious complication of epidural analgesia. Other potential complications include hypotension, decreased sensation and movement of the extremities, allergic reactions, and urine retention. Typically, epidural analgesia causes central nervous system depression (indicated by drowsiness) as well as a decreased heart rate and blood pressure.

At 11 p.m., a male client is admitted to the emergency department. He has a respiratory rate of 44 breaths/minute. He's anxious, and wheezes are audible. The client is immediately given oxygen by face mask and methylprednisolone (Depo-medrol) I.V. At 11:30 p.m., the client's arterial blood oxygen saturation is 86% and he's still wheezing. The nurse should plan to administer: A. Alprazolam (Xanax). B. Propranolol (Inderal) C. Morphine. D. Albuterol (Proventil).

D. The client is hypoxemic because of bronchoconstriction as evidenced by wheezes and a subnormal arterial oxygen saturation level. The client's greatest need is bronchodilation, which can be accomplished by administering bronchodilators. Albuterol is a beta2 adrenergic agonist, which causes dilation of the bronchioles. It's given by nebulization or metered-dose inhalation and may be given as often as every 30 to 60 minutes until relief is accomplished. Alprazolam is an anxiolytic and central nervous system depressant, which could suppress the client's breathing. Propranolol is contraindicated in a client who's wheezing because it's a beta2 adrenergic antagonist. Morphine is a respiratory center depressant and is contraindicated in this situation.

A female client is undergoing a complete physical examination as a requirement for college. When checking the client's respiratory status, the nurse observes respiratory excursion to help assess: A. Lung vibrations. B. Vocal sounds. C. Breath sounds. D. Chest movements.

D. The nurse observes respiratory excursion to help assess chest movements. Normally, thoracic expansion is symmetrical; unequal expansion may indicate pleural effusion, atelectasis, pulmonary embolus, or a rib or sternum fracture. The nurse assesses vocal sounds to evaluate air flow when checking for tactile fremitus; after asking the client to say "99," the nurse palpates the vibrations transmitted from the bronchopulmonary system along the solid surfaces of the chest wall to the nurse's palms. The nurse assesses breath sounds during auscultation.

A male adult patient hospitalized for treatment of a pulmonary embolism develops respiratory alkalosis. Which clinical findings commonly accompany respiratory alkalosis? A. Nausea or vomiting B. Abdominal pain or diarrhea C. Hallucinations or tinnitus D. Lightheadedness or paresthesia

D. The patient with respiratory alkalosis may complain of lightheadedness or paresthesia (numbness and tingling in the arms and legs). Nausea, vomiting, abdominal pain, and diarrhea may accompany respiratory acidosis. Hallucinations and tinnitus rare are associated with respiratory alkalosis or any other acid-base imbalance.

Nurse Ruth assessing a patient for tracheal displacement should know that the trachea will deviate toward the: A. Contralateral side in a simple pneumothorax B. Affected side in a hemothorax C. Affected side in a tension pneumothorax D. Contralateral side in hemothorax

D. The trachea will shift according to the pressure gradients within the thoracic cavity. In tension pneumothorax and hemothorax, accumulation of air or fluid causes a shift away from the injured side. If there is no significant air or fluid accumulation, the trachea will not shift. Tracheal deviation toward the contralateral side in simple pneumothorax is seen when the thoracic contents shift in response to the release of normal thoracic pressure gradients on the injured side.

A nurse is assisting a physician with the removal of a chest tube. The nurse should instruct the client to: A. Exhale slowly B. Stay very still C. Inhale and exhale quickly D. Perform the Valsalva maneuver

D. When the chest tube is removed, the client is asked to perform the Valsalva maneuver (take a deep breath, exhale, and bear down). The tube is quickly withdrawn, and an airtight dressing is taped in place. An alternative instruction is to ask the client to take a deep breath and hold the breath while the tube is removed. Options A, B, and C are incorrect client instructions.

When the patient is diagnosed with a lung abscess, what should the nurse teach the patient? Lobectomy surgery is usually needed to drain the abscess. IV antibiotic therapy will be used for a prolonged period of time. Oral antibiotics will be used when the patient and x-ray shows evidence of improvement. No further culture and sensitivity tests are needed if the patient takes the medication as ordered.

IV antibiotics are used until the patient and x-ray show evidence of improvement. Then oral antibiotics are used for a prolonged period of time. Culture and sensitivity testing is done during the course of antibiotic therapy to ensure that the infecting organism is not becoming resistant to the antibiotic as well as at the completion of the antibiotic therapy. Lobectomy surgery is only needed when reinfection of a large cavitary lesion occurs or to establish a diagnosis when there is evidence of a neoplasm or other underlying problem.

One week after a thoracotomy, a patient with chest tubes (CTs) to water-seal drainage has an air leak into the closed chest drainage system (CDS). Which patient assessment warrants follow-up nursing interventions? Water-seal chamber has 5 cm of water. No new drainage in collection chamber Chest tube with a loose-fitting dressing Small pneumothorax at CT insertion site

If the dressing at the CT insertion site is loose, an air leak will occur and will need to be sealed. The water-seal chamber usually has 2 cm of water, but having more water will not contribute to an air leak, and it should not be drained from the CDS. No new drainage does not indicate an air leak but may indicate the CT is no longer needed. If there is a pneumothorax, the chest tube should remove the air.

When planning teaching for the patient with chronic obstructive pulmonary disease (COPD), the nurse understands that what causes the manifestations of the disease? An overproduction of the antiprotease α1 -antitrypsin Hyperinflation of alveoli and destruction of alveolar walls Hypertrophy and hyperplasia of goblet cells in the bronchi Collapse and hypoventilation of the terminal respiratory unit

In COPD there are structural changes that include hyperinflation of alveoli, destruction of alveolar walls, destruction of alveolar capillary walls, narrowing of small airways, and loss of lung elasticity. An autosomal recessive deficiency of antitrypsin may cause COPD. Not all patients with COPD have excess mucus production by the increased number of goblet cells.

The nurse is teaching a patient how to self-administer ipratropium (Atrovent) via a metered dose inhaler (MDI). Which instruction given by the nurse is most appropriate to help the patient learn the proper inhalation technique? "Avoid shaking the inhaler before use." "Breathe out slowly before positioning the inhaler." "Using a spacer should be avoided for this type of medication." "After taking a puff, hold the breath for 30 seconds before exhaling."

It is important to breathe out slowly before positioning the inhaler. This allows the patient to take a deeper breath while inhaling the medication, thus enhancing the effectiveness of the dose. The inhaler should be shaken well. A spacer may be used. Holding the breath after the inhalation of medication helps keep the medication in the lungs, but 30 seconds will not be possible for a patient with COPD.

While teaching a patient with asthma about the appropriate use of a peak flow meter, what should the nurse instruct the patient to do? Keep a record of the peak flow meter numbers if symptoms of asthma are getting worse. Use the flow meter each morning after taking medications to evaluate their effectiveness. Increase the doses of the long-term control medication if the peak flow numbers decrease. Empty the lungs and then inhale quickly through the mouthpiece to measure how fast air can be inhaled.

It is important to keep track of peak flow readings daily, especially when the patient's symptoms are getting worse. The patient should have specific directions as to when to call the physician based on personal peak flow numbers. Peak flow is measured by exhaling into the flow meter and should be assessed before and after medications to evaluate their effectiveness.

The nurse is evaluating if a patient understands how to safely determine whether a metered dose inhaler (MDI) is empty. The nurse interprets that the patient understands this important information to prevent medication underdosing when the patient describes which method to check the inhaler? Place it in water to see if it floats. Keep track of the number of inhalations used. Shake the canister while holding it next to the ear Check the indicator line on the side of the canister.

It is no longer appropriate to see if a canister floats in water or not since this is not an accurate way to determine the remaining inhaler doses. The best method to determine when to replace an inhaler is by knowing the maximum puffs available per MDI and then replacing it after the number of days when those inhalations have been used. (100 puffs/2 puffs each day = 50 days)

A patient has been receiving oxygen per nasal cannula while hospitalized for COPD. The patient asks the nurse whether oxygen use will be needed at home. What is the most appropriate response by the nurse? "Long-term home oxygen therapy should be used to prevent respiratory failure." "Oxygen will not be needed until or unless you are in the terminal stages of this disease." "Long-term home oxygen therapy should be used to prevent heart problems related to COPD." "You will not need oxygen until your oxygen saturation drops to 88% and you have symptoms of hypoxia."

Long-term oxygen therapy in the home will not be considered until the oxygen saturation is less than or equal to 88% and the patient has signs of tissue hypoxia, such as cor pulmonale, erythrocytosis, or impaired mental status. PaO2 less than 55 mm Hg will also allow home oxygen therapy to be considered.

To promote airway clearance in a patient with pneumonia, what should the nurse instruct the patient to do (select all that apply)? Maintain adequate fluid intake. Splint the chest when coughing. Maintain a 30-degree elevation. Maintain a semi-Fowler's position. Instruct patient to cough at end of exhalation.*

Maintaining adequate fluid intake liquefies secretions, allowing easier expectoration. The nurse should instruct the patient to splint the chest while coughing. This will reduce discomfort and allow for a more effective cough. Coughing at the end of exhalation promotes a more effective cough. The patient should be positioned in an upright sitting position (high Fowler's) with head slightly flexed.

The patient with Parkinson's disease has a pulse oximetry reading of 72%, but he is not displaying any other signs of decreased oxygenation. What is most likely contributing to his low SpO2 level? Motion Anemia Dark skin color Thick acrylic nails

Motion is the most likely cause of the low SpO2 for this patient with Parkinson's disease. Anemia, dark skin color, and thick acrylic nails as well as low perfusion, bright fluorescent lights, and intravascular dyes may also cause an inaccurate pulse oximetry result. There is no mention of these or reason to suspect these in this question.

Which test result identifies that a patient with asthma is responding to treatment? An increase in CO2 levels A decreased exhaled nitric oxide A decrease in white blood cell count An increase in serum bicarbonate levels

Nitric oxide levels are increased in the breath of people with asthma. A decrease in the exhaled nitric oxide concentration suggests that the treatment may be decreasing the lung inflammation associated with asthma and adherence to treatment. An increase in CO2 levels, decreased white blood cell count, and increased serum bicarbonate levels do not indicate a positive response to treatment in the asthma patient.

During admission of a patient diagnosed with non-small cell lung carcinoma, the nurse questions the patient related to a history of which risk factors for this type of cancer (select all that apply)? Asbestos exposure Exposure to uranium Chronic interstitial fibrosis History of cigarette smoking Geographic area in which he was born

Non-small cell carcinoma is associated with cigarette smoking and exposure to environmental carcinogens, including asbestos and uranium. Chronic interstitial fibrosis is associated with the development of adenocarcinoma of the lung. Exposure to cancer-causing substances in the geographic area where the patient has lived for some time may be a risk, but not necessarily where the patient was born.

The nurse determines that a patient is experiencing common adverse effects from the inhaled corticosteroid beclomethasone (Beclovent) after what occurs? Hypertension and pulmonary edema Oropharyngeal candidiasis and hoarseness Elevation of blood glucose and calcium levels Adrenocortical dysfunction and hyperglycemia

Oropharyngeal candidiasis and hoarseness are common adverse effects from the use of inhaled corticosteroids because the medication can lead to overgrowth of organisms and local irritation if the patient does not rinse the mouth following each dose.

A patient with a recent history of a dry cough has had a chest x-ray that revealed the presence of nodules. In an effort to determine whether the nodules are malignant or benign, what is the primary care provider likely to order? Thoracentesis Pulmonary angiogram CT scan of the patient's chest Positron emission tomography (PET)

PET is used to distinguish benign and malignant pulmonary nodules. Because malignant lung cells have an increased uptake of glucose, the PET scan (which uses an IV radioactive glucose preparation) can demonstrate increased uptake of glucose in malignant lung cells. This differentiation cannot be made using CT, a pulmonary angiogram, or thoracentesis.

When the patient with a persisting cough is diagnosed with pertussis (instead of acute bronchitis), the nurse knows that treatment will include which type of medication? Antibiotic Corticosteroid Bronchodilator Cough suppressant

Pertussis, unlike acute bronchitis, is caused by a gram-negative bacillus, Bordella pertussis, which must be treated with antibiotics. Corticosteroids and bronchodilators are not helpful in reducing symptoms. Cough suppressants and antihistamines are ineffective and may induce coughing episodes with pertussis.

The patient had video-assisted thoracic surgery (VATS) to perform a lobectomy. What does the nurse know is the reason for using this type of surgery? The patient has lung cancer. The incision will be medial sternal or lateral. Chest tubes will not be needed postoperatively. Less discomfort and faster return to normal activity

The VATS procedure uses minimally invasive incisions that cause less discomfort and allow faster healing and return to normal activity as well as lower morbidity risk and fewer complications. Many surgeries can be done for lung cancer, but pneumonectomy via thoracotomy is the most common surgery for lung cancer. The incision for a thoracotomy is commonly a medial sternotomy or a lateral approach. A chest tube will be needed postoperatively for VATS.

A patient with an acute exacerbation of chronic obstructive pulmonary disease (COPD) needs to receive precise amounts of oxygen. Which equipment should the nurse prepare to use? Oxygen tent Venturi mask Nasal cannula Oxygen-conserving cannula

The Venturi mask delivers precise concentrations of oxygen and should be selected whenever this is a priority concern. The other methods are less precise in terms of amount of oxygen delivered.

When teaching the patient with chronic obstructive pulmonary disease (COPD) about smoking cessation, what information should be included related to the effects of smoking on the lungs and the increased incidence of pulmonary infections? Smoking causes a hoarse voice. Cough will become nonproductive. Decreased alveolar macrophage function Sense of smell is decreased with smoking.

The damage to the lungs includes alveolar macrophage dysfunction that increases the incidence of infections and thus increases patient discomfort and cost to treat the infections. Other lung damage that contributes to infections includes cilia paralysis or destruction, increased mucus secretion, and bronchospasms that lead to sputum accumulation and increased cough. The patient may already be aware of respiratory mucosa damage with hoarseness and decreased sense of smell and taste, but these do not increase the incidence of pulmonary infection.

The nurse evaluates that discharge teaching for a patient hospitalized with pneumonia has been effective when the patient makes which statement about measures to prevent a relapse? "I will seek immediate medical treatment for any upper respiratory infections." "I should continue to do deep-breathing and coughing exercises for at least 12 weeks." "I will increase my food intake to 2400 calories a day to keep my immune system well." "I must have a follow-up chest x-ray in 6 to 8 weeks to evaluate the pneumonia's resolution."

The follow-up chest x-ray will be done in 6 to 8 weeks to evaluate pneumonia resolution. A patient should seek medical treatment for upper respiratory infections that persist for more than 7 days. It may be important for the patient to continue with coughing and deep breathing exercises for 6 to 8 weeks, not 12 weeks, until all of the infection has cleared from the lungs. Increased fluid intake, not caloric intake, is required to liquefy secretions.

The nurse is assigned to care for a patient who has anxiety and an exacerbation of asthma. What is the primary reason for the nurse to carefully inspect the chest wall of this patient? Allow time to calm the patient. Observe for signs of diaphoresis. Evaluate the use of intercostal muscles. Monitor the patient for bilateral chest expansion.

The nurse physically inspects the chest wall to evaluate the use of intercostal (accessory) muscles, which gives an indication of the degree of respiratory distress experienced by the patient. The other options may also occur, but they are not the primary reason for inspecting the chest wall of this patient.

The patient seeks relief from the symptoms of an upper respiratory infection (URI) that has lasted for 5 days. Which patient assessment should the nurse use to help determine if the URI has developed into acute sinusitis? Coughing Fever, chills Dust allergy Maxillary pain

The nurse should assess the patient for sinus pain or pressure as a clinical indicator of acute sinusitis. Coughing and fever are nonspecific clinical indicators of a URI. A history of an allergy that is likely to affect the upper respiratory tract is supportive of the sinusitis diagnosis but is not specific for sinusitis.

After admitting a patient from home to the medical unit with a diagnosis of pneumonia, which physician orders will the nurse verify have been completed before administering a dose of cefuroxime (Ceftin) to the patient? Orthostatic blood pressures Sputum culture and sensitivity Pulmonary function evaluation Serum laboratory studies ordered for AM

The nurse should ensure that the sputum for culture and sensitivity was sent to the laboratory before administering the cefuroxime as this is community-acquired pneumonia. It is important that the organisms are correctly identified (by the culture) before the antibiotic takes effect. The test will also determine whether the proper antibiotic has been ordered (sensitivity testing). Although antibiotic administration should not be unduly delayed while waiting for the patient to expectorate sputum, orthostatic BP, pulmonary function evaluation, and serum laboratory tests will not be affected by the administration of antibiotics.

Which position is most appropriate for the nurse to place a patient experiencing an asthma exacerbation? Supine Lithotomy High Fowler's Reverse Trendelenburg

The patient experiencing an asthma attack should be placed in high Fowler's position and may need to lean forward to allow for optimal chest expansion and enlist the aid of gravity during inspiration. The supine, lithotomy, and reverse Trendelenburg positions will not facilitation ventilation.

When teaching the patient with cystic fibrosis about the diet and medications, what is the priority information to be included in the discussion? Fat soluble vitamins and dietary salt should be avoided. Insulin may be needed with a diabetic diet if diabetes mellitus develops. Pancreatic enzymes and adequate fat, calories, protein, and vitamins are needed. Distal intestinal obstruction syndrome (DIOS) can be treated with increased water.

The patient must take pancreatic enzymes before each meal and snack and adequate fat, calories, protein, and vitamins should be eaten. Fat-soluble vitamins are needed because they are malabsorbed with the excess mucus in the gastrointestinal system. Insulin may be needed, but there is no longer a diabetic diet, and this is not priority information at this time. DIOS develops in the terminal ileum and is treated with balanced polyethylene glycol electrolyte solution (MiraLAX) to thin bowel contents.

The physician has prescribed salmeterol (Serevent) for a patient with asthma. In reviewing the use of dry powder inhalers (DPIs) with the patient, what instructions should the nurse provide? "Close lips tightly around the mouthpiece and breathe in deeply and quickly." "To administer a DPI, you must use a spacer that holds the medicine so that you can inhale it." "You will know you have correctly used the DPI when you taste or sense the medicine going into your lungs." "Hold the inhaler several inches in front of your mouth and breathe in slowly, holding the medicine as long as possible."

The patient should be instructed to tightly close the lips around the mouthpiece and breathe in deeply and quickly to ensure the medicine moves down deeply into the lungs. Dry powder inhalers do not require spacer devices. The patient may not taste or sense the medicine going into the lungs.

Which statement made by the patient with chronic obstructive pulmonary disease (COPD) indicates a need for further teaching regarding the use of an ipratropium inhaler? "I can rinse my mouth following the two puffs to get rid of the bad taste." "I should wait at least 1 to 2 minutes between each puff of the inhaler." "Because this medication is not fast-acting, I cannot use it in an emergency if my breathing gets worse." "If my breathing gets worse, I should keep taking extra puffs of the inhaler until I can breathe more easily.

The patient should not just keep taking extra puffs of the inhaler to make breathing easier. Excessive treatment could trigger paradoxical bronchospasm, which would worsen the patient's respiratory status. Rinsing the mouth after the puffs will eliminate a bad taste. Waiting 1 to 2 minutes between each puff will facilitate the effectiveness of the administration. Ipratropium is not used in an emergency for COPD.

Before discharge, the nurse discusses activity levels with a 61-year-old patient with chronic obstructive pulmonary disease (COPD) and pneumonia. Which exercise goal is most appropriate once the patient is fully recovered from this episode of illness? Slightly increase activity over the current level. Swim for 10 min/day, gradually increasing to 30 min/day. Limit exercise to activities of daily living to conserve energy. Walk for 20 min/day, keeping the pulse rate less than 130 beats/min.

The patient will benefit from mild aerobic exercise that does not stress the cardiorespiratory system. The patient should be encouraged to walk for 20 min/day, keeping the pulse rate less than 75% to 80% of maximum heart rate (220 - patient's age).

The patient with HIV has been diagnosed with Candida albicans, an opportunistic infection. The nurse knows the patient needs more teaching when she says, "I will be given amphotericin B to treat the fungus." "I got this fungus because I am immunocompromised." "I need to be isolated from my family and friends so they won't get it." "The effectiveness of my therapy can be monitored with fungal serology titers."

The patient with an opportunistic fungal infection does not need to be isolated because it is not transmitted from person to person. This immunocompromised patient will be likely to have a serious infection so it will be treated with IV amphotericin B. The effectiveness of the therapy can be monitored with fungal serology titers.

When assessing a patient's sleep-rest pattern related to respiratory health, what should the nurse ask the patient about (select all that apply)? Have trouble falling asleep? Need to urinate during the night? Awaken abruptly during the night? Sleep more than 8 hours per night? Need to sleep with the head elevated?

The patient with sleep apnea may have insomnia and/or abrupt awakenings. Patients with cardiovascular disease (e.g., heart failure that may affect respiratory health) may need to sleep with the head elevated on several pillows (orthopnea). Sleeping more than 8 hours per night or needing to urinate during the night is not indicative of impaired respiratory health.

The nurse is caring for a 73-year-old patient who underwent a left total knee arthroplasty. On the third postoperative day, the patient complains of shortness of breath, slight chest pain, and that "something is wrong." Temperature is 98.4° F, blood pressure 130/88, respirations 36/minute, and oxygen saturation 91% on room air. What action should the nurse take first? Notify the physician. Administer a nitroglycerin tablet sublingually. Conduct a thorough assessment of the chest pain. Sit the patient up in bed as tolerated and apply oxygen.

The patient's clinical picture is most likely pulmonary embolus, and the first action the nurse takes should be to assist with the patient's respirations. For this reason, the nurse should sit the patient up as tolerated and apply oxygen before notifying the physician. The nitroglycerin tablet would not be helpful, and the oxygenation status is a bigger problem than the slight chest pain at this time.

A male patient with COPD becomes dyspneic at rest. His baseline blood gas results are PaO2 70 mm Hg, PaCO2 52 mm Hg, and pH 7.34. What updated patient assessment requires the nurse's priority intervention? Arterial pH 7.26 PaCO2 50 mm Hg Patient in tripod position Increased sputum expectoration

The patient's pH shows acidosis that supports an exacerbation of COPD along with the worsening dyspnea. The PaCO2 has improved from baseline, the tripod position helps the patient's breathing, and the increase in sputum expectoration will improve the patient's ventilation.

During discharge teaching for a 65-year-old patient with chronic obstructive pulmonary disease (COPD) and pneumonia, which vaccine should the nurse recommend that this patient receive? Pneumococcal Staphylococcus aureus Haemophilus influenzae Bacille-Calmette-Guérin (BCG)

The pneumococcal vaccine is important for patients with a history of heart or lung disease, recovering from a severe illness, age 65 or over, or living in a long-term care facility. A Staphylococcus aureus vaccine has been researched but not yet been effective. The Haemophilus influenzae vaccine would not be recommended as adults do not need it unless they are immunocompromised. The BCG vaccine is for infants in parts of the world where tuberculosis (TB) is prevalent.

Which physical assessment finding in a patient with a lower respiratory problem best supports the nursing diagnosis of ineffective airway clearance? Basilar crackles Respiratory rate of 28 Oxygen saturation of 85% Presence of greenish sputum

The presence of adventitious breath sounds indicates that there is accumulation of secretions in the lower airways. This would be consistent with a nursing diagnosis of ineffective airway clearance because the patient is retaining secretions. The rapid respiratory rate, low oxygen saturation, and presence of greenish sputum may occur with a lower respiratory problem, but do not definitely support the nursing diagnosis of ineffective airway clearance.

The nurse is caring for a patient with an acute exacerbation of asthma. Following initial treatment, what finding indicates to the nurse that the patient's respiratory status is improving? Wheezing becomes louder. Cough remains nonproductive. Vesicular breath sounds decrease. Aerosol bronchodilators stimulate coughing.

The primary problem during an exacerbation of asthma is narrowing of the airway and subsequent diminished air exchange. As the airways begin to dilate, wheezing gets louder because of better air exchange. Vesicular breath sounds will increase with improved respiratory status. After a severe asthma exacerbation, the cough may be productive and stringy. Coughing after aerosol bronchodilators may indicate a problem with the inhaler or its use.

What is the priority nursing assessment in the care of a patient who has a tracheostomy? Electrolyte levels and daily weights Assessment of speech and swallowing Respiratory rate and oxygen saturation Pain assessment and assessment of mobility

The priority assessment in the care of a patient with a tracheostomy focuses on airway and breathing. These assessments supersede the nurse's assessments that may also be necessary, such as nutritional status, speech, pain, and swallowing ability.

The nurse teaches pursed lip breathing to a patient who is newly diagnosed with chronic obstructive pulmonary disease (COPD). The nurse reinforces that this technique will assist respiration by which mechanism? Loosening secretions so that they may be coughed up more easily Promoting maximal inhalation for better oxygenation of the lungs Preventing bronchial collapse and air trapping in the lungs during exhalation Increasing the respiratory rate and giving the patient control of respiratory patterns

The purpose of pursed lip breathing is to slow down the exhalation phase of respiration, which decreases bronchial collapse and subsequent air trapping in the lungs during exhalation. It does not affect secretions, inhalation, or increase the rate of breathing.

When teaching the patient with bronchiectasis about manifestations to report to the health care provider, which manifestation should be included? Increasing dyspnea Temperature below 98.6° F Decreased sputum production Unable to drink 3 L low-sodium fluids

The significant clinical manifestations to report to the health care provider include increasing dyspnea, fever, chills, increased sputum production, bloody sputum, and chest pain. Although drinking at least 3 L of low-sodium fluid will help liquefy secretions to make them easier to expectorate, the health care provider does not need to be notified if the patient cannot do this one day.

The patient is calling the clinic with a cough. What assessment should be made first before the nurse advises the patient? Cough sound, sputum production, pattern Frequency, a family history, hematemesis Smoking, medications, residence location Weight loss, activity tolerance, orthopnea

The sound of the cough, sputum production and description, as well as pattern of the cough's occurrence (including acute or chronic) and what its occurrence is related to are the first assessments to be made to determine the severity. Frequency of the cough will not provide a lot of information. Family history can help to determine a genetic cause of the cough. Hematemesis is vomiting blood and not as important as hemoptysis. Smoking is an important risk factor for COPD and lung cancer and may cause a cough. Medications may or may not contribute to a cough as does residence location. Weight loss, activity intolerance, and orthopnea may be related to respiratory or cardiac problems, but are not as important when dealing with a cough.

The patient has decided to use the voice rehabilitation that offers the best speech quality even though it must be cleaned regularly. The nurse knows that this is what kind of voice rehabilitation? Electromyograph Intraoral electrolarynx Neck type electrolarynx Transesophageal puncture

The transesophageal puncture provides a fistula between the esophagus and trachea with a one-way valved prosthesis to prevent aspiration from the esophagus to the trachea. Air moves from the lungs, vibrates against the esophagus, and words are formed with the tongue and lips as the air moves out the mouth. The electromyography and both electrolarynx methods produce low-pitched mechanical sounds.

What nursing intervention is most appropriate to enhance oxygenation in a patient with unilateral malignant lung disease? Positioning patient on right side Maintaining adequate fluid intake Positioning patient with "good lung" down Performing postural drainage every 4 hours

Therapeutic positioning identifies the best position for the patient, thus assuring stable oxygenation status. Research indicates that positioning the patient with the unaffected lung (good lung) dependent best promotes oxygenation in patients with unilateral lung disease. For bilateral lung disease, the right lung down has best ventilation and perfusion. Increasing fluid intake and performing postural drainage will facilitate airway clearance, but positioning is most appropriate to enhance oxygenation.

After swallowing, a 73-year-old patient is coughing and has a wet voice. What changes of aging could be contributing to this abnormality? Decreased response to hypercapnia Decreased number of functional alveoli Increased calcification of costal cartilage Decreased respiratory defense mechanisms

These manifestations are associated with aspiration, which more easily occur in the right lung as the right mainstem bronchus is shorter, wider, and straighter than the left mainstem bronchus. Aspiration occurs more easily in the older patient related to decreased respiratory defense mechanisms (e.g., decreases in immunity, ciliary function, cough force, sensation in pharynx). Changes of aging include a decreased response to hypercapnia, decreased number of functional alveoli, and increased calcification of costal cartilage, but these do not increase the risk of aspiration.

During the assessment in the ED, the nurse is palpating the patient's chest. Which finding is a medical emergency? Trachea moved to the left Increased tactile fremitus Decreased tactile fremitus Diminished chest movement

Tracheal deviation is a medical emergency when it is caused by a tension pneumothorax. Tactile fremitus increases with pneumonia or pulmonary edema and decreases in pleural effusion or lung hyperinflation. Diminished chest movement occurs with barrel chest, restrictive disease, and neuromuscular disease.

In assessment of the patient with acute respiratory distress, what should the nurse expect to observe (select all that apply)? Cyanosis Tripod position Kussmaul respirations Accessory muscle use Increased AP diameter

Tripod position and accessory muscle use indicate moderate to severe respiratory distress. Cyanosis may be related to anemia, decreased oxygen transfer in the lungs, or decreased cardiac output. Therefore it is a nonspecific and unreliable indicator of only respiratory distress. Kussmaul respirations occur when the patient is in metabolic acidosis to increase CO2 excretion. Increased AP diameter occurs with lung hyperinflation from COPD, cystic fibrosis, or with advanced age.

When the patient is experiencing metabolic acidosis secondary to type 1 diabetes mellitus, what physiologic response should the nurse expect to assess in the patient? Vomiting Increased urination Decreased heart rate Rapid respiratory rate

When a patient with type 1 diabetes has hyperglycemia and ketonemia causing metabolic acidosis, the physiologic response is to increase the respiratory rate and tidal volume to blow off the excess CO2. Vomiting and increased urination may occur with hyperglycemia, but not as physiologic responses to metabolic acidosis. The heart rate will increase.

When initially teaching a patient the supraglottic swallow following a radical neck dissection, with which food or fluid should the nurse begin? Cola Applesauce French fries White grape juice

When learning the supraglottic swallow, it may be helpful to start with carbonated beverages because the effervescence provides clues about the liquid's position. Thin, watery fluids should be avoided because they are difficult to swallow and increase the risk of aspiration. Nonpourable pureed foods, such as applesauce, would decrease the risk of aspiration, but carbonated beverages are the better choice with which to start.

The nurse identifies the nursing diagnosis of activity intolerance for a patient with asthma. In patients with asthma, the nurse assesses for which etiologic factor for this nursing diagnosis? Work of breathing Fear of suffocation Effects of medications Anxiety and restlessness

When the patient does not have sufficient gas exchange to engage in activity, the etiologic factor is often the work of breathing. When patients with asthma do not have effective respirations, they use all available energy to breathe and have little left over for purposeful activity. Fear of suffocation, effects of medications or anxiety, and restlessness are not etiologies for activity intolerance for a patient with asthma.

The patient's arterial blood gas results show the PaO2 at 65 mmHg and the SaO2 at 80%. What early manifestations should the nurse expect to observe in this patient? Restlessness, tachypnea, tachycardia, and diaphoresis Unexplained confusion, dyspnea at rest, hypotension, and diaphoresis Combativeness, retractions with breathing, cyanosis, and decreased output Coma, accessory muscle use, cool and clammy skin, and unexplained fatigue

With inadequate oxygenation, early manifestations include restlessness, tachypnea, tachycardia, and diaphoresis, decreased urinary output, and unexplained fatigue. The unexplained confusion, dyspnea at rest, hypotension, and diaphoresis; combativeness, retractions with breathing, cyanosis, and decreased urinary output; coma, accessory muscle use, cool and clammy skin, and unexplained fatigue occur as later manifestations of inadequate oxygenation.


Conjuntos de estudio relacionados

Module 9 - Obsessive and Compulsive Related Disorders

View Set

Drugs & Behavior - Test 4 - Chapter 7 & 8

View Set

Quiz: Put on Sterile Gloves and Remove Soiled Glove

View Set

Week 2: Stakeholders & Workers' Co-Operative

View Set

Entrepreneurship II Final Exam Review

View Set

GDP, unemployment, and inflation test review

View Set

Chapter 16/17 US history study guide

View Set